\( \def\<#1>{\left<#1\right>} \let\geq\geqslant \let\leq\leqslant % an undirected version of \rightarrow: \newcommand{\mathdash}{\relbar\mkern-9mu\relbar} \def\deg#1{\mathrm{deg}(#1)} \newcommand{\dg}[1]{d_{#1}} \newcommand{\Norm}{\mathrm{N}} \newcommand{\const}[1]{c_{#1}} \newcommand{\cconst}[1]{\alpha_{#1}} \newcommand{\Exp}[1]{E_{#1}} \newcommand*{\ppr}{\mathbin{\ensuremath{\otimes}}} \newcommand*{\su}{\mathbin{\ensuremath{\oplus}}} \newcommand{\nulis}{\vmathbb{0}} %{\mathbf{0}} \newcommand{\vienas}{\vmathbb{1}} \newcommand{\Up}[1]{#1^{\uparrow}} %{#1^{\vartriangle}} \newcommand{\Down}[1]{#1^{\downarrow}} %{#1^{\triangledown}} \newcommand{\lant}[1]{#1_{\mathrm{la}}} % lower antichain \newcommand{\uant}[1]{#1_{\mathrm{ua}}} % upper antichain \newcommand{\skal}[1]{\langle #1\rangle} \newcommand{\NN}{\mathbb{N}} % natural numbers \newcommand{\RR}{\mathbb{R}} \newcommand{\minTrop}{\mathbb{T}_{\mbox{\rm\footnotesize min}}} \newcommand{\maxTrop}{\mathbb{T}_{\mbox{\rm\footnotesize max}}} \newcommand{\FF}{\mathbb{F}} \newcommand{\pRR}{\mathbb{R}_{\mbox{\tiny $+$}}} \newcommand{\QQ}{\mathbb{Q}} \newcommand{\ZZ}{\mathbb{Z}} \newcommand{\gf}[1]{GF(#1)} \newcommand{\conv}[1]{\mathrm{Conv}(#1)} \newcommand{\vvec}[2]{\vec{#1}_{#2}} \newcommand{\f}{{\mathcal F}} \newcommand{\h}{{\mathcal H}} \newcommand{\A}{{\mathcal A}} \newcommand{\B}{{\mathcal B}} \newcommand{\C}{{\mathcal C}} \newcommand{\R}{{\mathcal R}} \newcommand{\MPS}[1]{f_{#1}} % matrix multiplication \newcommand{\ddeg}[2]{\#_{#2}(#1)} \newcommand{\length}[1]{|#1|} \DeclareMathOperator{\support}{sup} \newcommand{\supp}[1]{\support(#1)} \DeclareMathOperator{\Support}{sup} \newcommand{\spp}{\Support} \newcommand{\Supp}[1]{\mathrm{Sup}(#1)} %{\mathcal{S}_{#1}} \newcommand{\lenv}[1]{\lfloor #1\rfloor} \newcommand{\henv}[1]{\lceil#1\rceil} \newcommand{\homm}[2]{{#1}^{\langle #2\rangle}} \let\daug\odot \let\suma\oplus \newcommand{\compl}[1]{Y_{#1}} \newcommand{\pr}[1]{X_{#1}} \newcommand{\xcompl}[1]{Y'_{#1}} \newcommand{\xpr}[1]{X'_{#1}} \newcommand{\cont}[1]{A_{#1}} % content \def\fontas#1{\mathsf{#1}} %{\mathrm{#1}} %{\mathtt{#1}} % \newcommand{\arithm}[1]{\fontas{Arith}(#1)} \newcommand{\Bool}[1]{\fontas{Bool}(#1)} \newcommand{\linBool}[1]{\fontas{Bool}_{\mathrm{lin}}(#1)} \newcommand{\rBool}[2]{\fontas{Bool}_{#2}(#1)} \newcommand{\BBool}[2]{\fontas{Bool}_{#2}(#1)} \newcommand{\MMin}[1]{\fontas{Min}(#1)} \newcommand{\MMax}[1]{\fontas{Max}(#1)} \newcommand{\negMin}[1]{\fontas{Min}^{-}(#1)} \newcommand{\negMax}[1]{\fontas{Max}^{-}(#1)} \newcommand{\Min}[2]{\fontas{Min}_{#2}(#1)} \newcommand{\Max}[2]{\fontas{Max}_{#2}(#1)} \newcommand{\convUn}[1]{\fontas{L}_{\ast}(#1)} \newcommand{\Un}[1]{\fontas{L}(#1)} \newcommand{\kUn}[2]{\fontas{L}_{#2}(#1)} \newcommand{\Nor}{\mu} % norm without argument \newcommand{\nor}[1]{\Nor(#1)} \newcommand{\bool}[1]{\hat{#1}} % Boolean version of f \newcommand{\bphi}{\phi} % boolean circuit \newcommand{\xf}{\boldsymbol{\mathcal{F}}} \newcommand{\euler}{\mathrm{e}} \newcommand{\ee}{f} % other element \newcommand{\exchange}[3]{{#1}-{#2}+{#3}} \newcommand{\dist}[2]{{#2}[#1]} \newcommand{\Dist}[1]{\mathrm{dist}(#1)} \newcommand{\mdist}[2]{\dist{#1}{#2}} % min-max dist. \newcommand{\matching}{\mathcal{M}} \renewcommand{\E}{A} \newcommand{\F}{\mathcal{F}} \newcommand{\set}{W} \newcommand{\Deg}[1]{\mathrm{deg}(#1)} \newcommand{\mtree}{MST} \newcommand{\stree}{{\cal T}} \newcommand{\dstree}{\vec{\cal T}} \newcommand{\Rich}{U_0} \newcommand{\Prob}[1]{\ensuremath{\mathrm{Pr}\left\{{#1}\right\}}} \newcommand{\xI}{\boldsymbol{I}} \newcommand{\plus}{\mbox{\tiny $+$}} \newcommand{\sgn}[1]{\left[#1\right]} \newcommand{\ccompl}[1]{{#1}^*} \newcommand{\contr}[1]{[#1]} \newcommand{\harm}[2]{{#1}\,\#\,{#2}} %{{#1}\,\oplus\,{#2}} \newcommand{\hharm}{\#} %{\oplus} \newcommand{\rec}[1]{1/#1} \newcommand{\rrec}[1]{{#1}^{-1}} \DeclareRobustCommand{\bigO}{% \text{\usefont{OMS}{cmsy}{m}{n}O}} \newcommand{\dalyba}{/}%{\oslash} \newcommand{\mmax}{\mbox{\tiny $\max$}} \newcommand{\thr}[2]{\mathrm{Th}^{#1}_{#2}} \newcommand{\rectbound}{h} \newcommand{\pol}[3]{\sum_{#1\in #2}{#3}_{#1}\prod_{i=1}^n x_i^{#1_i}} \newcommand{\tpol}[2]{\min_{#1\in #2}\left\{\skal{#1,x}+\const{#1}\right\}} \newcommand{\comp}{\circ} % composition \newcommand{\0}{\vec{0}} \newcommand{\drops}[1]{\tau(#1)} \newcommand{\HY}[2]{F^{#2}_{#1}} \newcommand{\hy}[1]{f_{#1}} \newcommand{\hh}{h} \newcommand{\hymin}[1]{f_{#1}^{\mathrm{min}}} \newcommand{\hymax}[1]{f_{#1}^{\mathrm{max}}} \newcommand{\ebound}[2]{\partial_{#2}(#1)} \newcommand{\Lpure}{L_{\mathrm{pure}}} \newcommand{\Vpure}{V_{\mathrm{pure}}} \newcommand{\Lred}{L_1} %L_{\mathrm{red}}} \newcommand{\Lblue}{L_0} %{L_{\mathrm{blue}}} \newcommand{\epr}[1]{z_{#1}} \newcommand{\wCut}[1]{w(#1)} \newcommand{\cut}[2]{w_{#2}(#1)} \newcommand{\Length}[1]{l(#1)} \newcommand{\Sup}[1]{\mathrm{Sup}(#1)} \newcommand{\ddist}[1]{d_{#1}} \newcommand{\sym}[2]{S_{#1,#2}} \newcommand{\minsum}[2]{\mathrm{MinS}^{#1}_{#2}} \newcommand{\maxsum}[2]{\mathrm{MaxS}^{#1}_{#2}} % top k-of-n function \newcommand{\cirsel}[2]{\Phi^{#1}_{#2}} % its circuit \newcommand{\sel}[2]{\sym{#1}{#2}} % symmetric pol. \newcommand{\cf}[1]{{#1}^{o}} \newcommand{\Item}[1]{\item[\mbox{\rm (#1)}]} % item in roman \newcommand{\bbar}[1]{\underline{#1}} \newcommand{\Narrow}[1]{\mathrm{Narrow}(#1)} \newcommand{\Wide}[1]{\mathrm{Wide}(#1)} \newcommand{\eepsil}{\varepsilon} \newcommand{\amir}{\varphi} \newcommand{\mon}[1]{\mathrm{mon}(#1)} \newcommand{\mmon}{\alpha} \newcommand{\gmon}{\alpha} \newcommand{\hmon}{\beta} \newcommand{\nnor}[1]{\|#1\|} \newcommand{\inorm}[1]{\left\|#1\right\|_{\mbox{\tiny $\infty$}}} \newcommand{\mstbound}{\gamma} \newcommand{\coset}[1]{\textup{co-}{#1}} \newcommand{\spol}[1]{\mathrm{ST}_{#1}} \newcommand{\cayley}[1]{\mathrm{C}_{#1}} \newcommand{\SQUARE}[1]{\mathrm{SQ}_{#1}} \newcommand{\STCONN}[1]{\mathrm{STCON}_{#1}} \newcommand{\STPATH}[1]{\mathrm{PATH}_{#1}} \newcommand{\SSSP}[1]{\mathrm{SSSP}(#1)} \newcommand{\APSP}[1]{\mathrm{APSP}(#1)} \newcommand{\MP}[1]{\mathrm{MP}_{#1}} \newcommand{\CONN}[1]{\mathrm{CONN}_{#1}} \newcommand{\PERM}[1]{\mathrm{PER}_{#1}} \newcommand{\mst}[2]{\tau_{#1}(#2)} \newcommand{\MST}[1]{\mathrm{MST}_{#1}} \newcommand{\MIS}{\mathrm{MIS}} \newcommand{\dtree}{\mathrm{DST}} \newcommand{\DST}[1]{\dtree_{#1}} \newcommand{\CLIQUE}[2]{\mathrm{CL}_{#1,#2}} \newcommand{\ISOL}[1]{\mathrm{ISOL}_{#1}} \newcommand{\POL}[1]{\mathrm{POL}_{#1}} \newcommand{\ST}[1]{\ptree_{#1}} \newcommand{\Per}[1]{\mathrm{per}_{#1}} \newcommand{\PM}{\mathrm{PM}} \newcommand{\error}{\epsilon} \newcommand{\PI}[1]{A_{#1}} \newcommand{\Low}[1]{A_{#1}} \newcommand{\node}[1]{v_{#1}} \newcommand{\BF}[2]{W_{#2}[#1]} % Bellman-Ford \newcommand{\FW}[3]{W_{#1}[#2,#3]} % Floyd-Washall \newcommand{\HK}[1]{W[#1]} % Held-Karp \newcommand{\WW}[1]{W[#1]} \newcommand{\pWW}[1]{W^{+}[#1]} \newcommand{\nWW}[1]{W^-[#1]} \newcommand{\knap}[2]{W_{#2}[#1]} \newcommand{\Cut}[1]{w(#1)} \newcommand{\size}[1]{\mathrm{size}(#1)} \newcommand{\dual}[1]{{#1}^{\ast}} \def\gcd#1{\mathrm{gcd}(#1)} \newcommand{\econt}[1]{C_{#1}} \newcommand{\xecont}[1]{C_{#1}'} \newcommand{\rUn}[1]{\fontas{L}_{r}(#1)} \newcommand{\copath}{\mathrm{co}\text{-}\mathrm{Path}_n} \newcommand{\Path}{\mathrm{Path}_n} \newcommand{\slice}[2]{Q^{#1}_{#2}} \)
[This is a supplementary material to Section 4.2.4 of Chapter 4]

Approximating hypergraph matchings by (max,+) circuits is hard

As shown in Section 4.2.4 of the book, the maximization problem on any polynomial $(m,d)$-design $\f\subseteq 2^{[n]}$ requires large $(\max,+)$ circuits to approximate it within a factor $r$ twice smaller than $m/d$ (Corollary 4.24), but can be easily approximated within the factor $r=m/d$ (Proposition 4.23). This does not show that the greedy algorithm can outperform approximating $(\max,+)$ circuits, because the greedy algorithm cannot achieve any smaller than $r=m/d$ approximation factor for this problem.

To show this, take $\epsilon>0$ arbitrarily small, and set $c:=1/(1-\epsilon/2)>1$. Take arbitrary two sets $A\neq B\in\f$, and a subset $S\subset A$ of $|S|=d$ elements. Since $\f$ is an $(m,d)$-design, $S$ cannot be contained in $B$. So, give weight $c>1$ to all elements of $S$, weight $1$ to all elements of $B\setminus S$, and zero weight to the rest. Then the maximizing (best-in) greedy algorithm picks elements of weight $c$ first, gets all $|S|=d$ of them, but then is stuck because no element of weight $1$ fits; hence, the greedy algorithm achieves the total weight $c|S|=c d$. But the optimum is at least $|B|=m$. Hence, the approximation factor is at least $m/c d=(1-\epsilon/2)m/d >(1-\epsilon)m/d$.     $\Box$

More on greedy algorithms can be found in this comment.

To show that the greedy algorithm can still outperform approximating $(\max,+)$ circuits, another maximization problem was considered in [1]: maximum weight perfect matchings in a complete $k$-partite hypergraph $K_{m,\ldots,m}$ with $m$ vertices in each part. (In the literature, such a graph is also called complete $k$-partite $k$-uniform hypergraph: "$k$-uniform" because each (hyper-) edge has exactly $k$ vertices, and "$k$-partite" because we have $k$ parts.)

In $K_{m,\ldots,m}$, we have a set $V=V_1\cup \cdots\cup V_k$ of $|V|=mk$ vertices decomposed into $k$ disjoint blocks $V_1,\ldots,V_k$, each of size $m$. Edges (called also hyperedges) are $k$-tuples $e\in V_1\times \cdots\times V_k$. The ground set \[ E=V_1\times \cdots\times V_k \] consists of all $|E|=m^k$ edges. Two edges are disjoint if they differ in all $k$ positions. A matching is a set of disjoint edges, and is a perfect matching if it has the maximum possible number $m$ of edges.

Hypergraph
Fig 1: A $k$-partite $k$-uniform hypergraph for $k=4$ with $m=5$ vertices in each block (part), and a perfect matching in it.

The family $\f_{m,k}$ of feasible solutions of our problem consists of all $|\f_{m,k}|=(m!)^{k-1}$ perfect matchings. So, the maximization problem on $\f_{m,k}$ is, given an assignment of nonnegative weights $x_e$ to the edges $e\in E$, to compute the maximum total weight \[ f(x)=\max\left\{x_{e_1}+\cdots+x_{e_m}\colon e_i\in E\,, \mbox{ and $e_i$ and $e_j$ are disjoint for all $i\neq j$ }\right\} \] of a perfect matching. Note that in the case $k=2$, $\f_{m,k}$ consists of perfect matchings in $K_{m,m}$, and the problem is to compute the maximum weight of such a perfect matching.

The greedy algorithm can approximate the maximization problem on $\f_{m,k}$ within the factor $k$ by just always picking the heaviest of the remaining edges, untouched by the partial matching picked so far. On the other hand, we have the following lower bound for $(\max,+)$ circuits approximating this problem.

Theorem A: Let $m$ be a sufficiently large integer, and $k=k(m)$ be an integer such that $6\leq k\leq \log \sqrt{m}$. If the allowed approximation factor is $r\leq 2^k/9$, then \[ \Max{\f_{m,k}}{r}=2^{\Omega(\sqrt{m})}\,. \]
Note that the lower bound holds even if the allowed approximation factor $r$ for $(\max,+)$ circuits is exponential in the approximation factor $k$ achieved by the greedy algorithm.

Proof: As done in Section 4.2.4 of the book in the case of maximization problems on designs, we will use the rectangle bound (Lemma 4.20(1)), but this time with the balance parameter $\gamma:=2/3$ (not $\gamma=2$) to have $\gamma/2=1-\gamma$.

So, take an arbitrary rectangle $\R=\A\lor \B$ lying below $\f=\f_{m,k}$ (this means that every set of $\R$ is contained in at least one set of $\f$). Hence, sets in $\A$ and in $\B$ are subsets of (hyper-)edges $e\in V_1\times \cdots\times V_k$. Since $\R$ lies below our family $\f$, and $\f$ consist of (perfect) matchings, all sets $A\cup B$ with $A\in\A$ and $B\in\B$ must also be matchings.

Take the integer $d:=\lceil m/3r\rceil$. Each perfect matching $F\in \f=\f_{m,k}$ has size $|F|=m$ (consists of $m$ disjoint edges). Note that for $\gamma=2/3$, we have that both $\tfrac{\gamma}{2}$ and $1-\gamma$ are equal to $1/3$. Hence, the inequalities right after Eq. (4.2) in this(1) footnote turn into: \[ |F\cap A|\geq \frac{\gamma}{2r}\cdot |F| = \frac{m}{3r}=d\ \ \mbox{ and }\ \ |F\cap B|\geq \frac{1-\gamma}{r}\cdot |F|=\frac{m}{3r}=d\,. \] By Lemma 4.20(1), it is enough to show that $|\f|/|\f_{\R}|=2^{\Omega(\sqrt{m})}$ holds for the family of perfect matchings: \[ \f_{\R}=\{F\in\f\colon \mbox{$|F\cap A|\geq d$ and $|F\cap B|\geq d$ for some $A\in\A$ and $B\in\B$ }\}\,. \]

Let $S\subseteq V$ be the set of vertices belonging to at least one edge of a matching in $\A$, and $T\subseteq V$ be the set of vertices belonging to at least one edge of a matching in $\B$. Since the rectangle $\R=\A\lor \B$ is cross-disjoint, we know that the matchings $A\in\A$ and $B\in\B$ must be edge-disjoint, that is, $A\cap B=\emptyset$ must hold. However, since the sets $A\cup B$ are matchings (the rectangle $\R$ lies below the family $\f$ of all perfect matchings), we actually know that matchings $A$ and $B$ are even vertex-disjoint. Thus, we have a crucial property: \[ S\cap T=\emptyset\,. \] Call a matching $A\subseteq V_1\times\cdots\times V_k$ an $S$-matching if $A\subseteq (V_1\cap S)\times\cdots\times (V_k\cap S)$ holds, that is, if edges of $A$ only match vertices of $S$; $T$-matchings are defined similarly. By the definition of $\f_{\R}$, every perfect matching $F\in\f_{\R}$ has at least $d$ edges lying in $(V_1\cap S)\times\cdots\times (V_k\cap S)$, and at least $d$ edges lying in $(V_1\cap T)\times\cdots\times (V_k\cap T)$. In particular, every perfect matching $F\in\f_{\R}$ must contain at least one matching $A\cup B$, where $A$ is an $S$-matching with $|A|=d$ edges and $B$ is a $T$-matching with $|B|=d$ edges. It therefore suffices to upper-bound the number of perfect matchings $F\in\f$ with this property.

We can pick any such pair $(A,B)$ of matchings as follows. Let $S_i=S\cap V_i$ and $T_i=T\cap V_i$ for $i=1,\ldots,k$. We can assume that each of these $2k$ sets has at least $d$ vertices, for otherwise none of the $S$-matchings or of the $T$-matchings could have $\geq d$ edges, implying that $\f_{\R}=\emptyset$.

  1. Pick in each $S_i$ a subset $S_i'\subseteq S_i$ of $|S_i'|=d$ vertices, and in each $T_i$ a subset $T_i'\subseteq T_i$ of $|T_i'|=d$ vertices. There are at most \[ \prod_{i=1}^k\binom{m_i}{d}\binom{m-m_i}{d} \leq \binom{m}{2d}^k \] possibilities to do this, where $m_i=|S_i|$.

  2. Pick a perfect matching $A$ in $S_1'\times \cdots \times S_k'$ and a perfect matching $B$ in $T_1'\times \cdots \times T_k'$. There are only $\left[(d!)^{k-1}\right]^2=(d!)^{2(k-1)}$ possibilities to do this.

After a pair $(A,B)$ of matchings is picked, there are at most $[(m-2d)!]^{k-1}$ possibilities to extend the matching $A\cup B$ to a perfect matching. Thus, \[ |\f_{\R}| \leq \binom{m}{2d}^{k} (d!)^{2(k-1)} [(m-2d)!]^{k-1} % = \binom{m}{2d}\left[m!\cdot \frac{(d!)^2}{(2d)!} \right]^{k-1} = \binom{m}{2d}\left[m!\cdot \binom{2d}{d}^{-1} \right]^{k-1}\,, \] where the equality follows because $\binom{m}{2d}=m!/(2d)!(m-2d)!$ and $(2d)!/(d!)^2=\binom{2d}{d}$. Since there are $|\f|=(m!)^{k-1}$ perfect matchings, the rectangle bound (Lemma 4.20(1) ) yields the following lower bound on $t=\Max{\f}{r}$: \begin{equation}\label{eq:hyper} t \geq \frac{|\f|}{|\f_{\R}|} \geq \frac{\binom{2d}{d}^{k-1}}{\binom{m}{2d}} \geq \left(\frac{2^{2d}}{d}\right)^{k-1}\cdot \left(\frac{2d}{\euler m}\right)^{2d}=\frac{1}{d^{k-1}}\left(\frac{2^kd}{\euler m}\right)^{2d} \geq \frac{1}{d^{k-1}}\left(\frac{2^k}{3\euler r}\right)^{2d}\,, \end{equation} where the second inequality follows from the inequalities $\binom{m}{2d}\leq (\euler m/2d)^{2d}$ and $\binom{2d}{d}\geq 2^{2d}/\sqrt{4d}\geq 2^{2d}/d$, and the last inequality follows because (by our choice) $d=\lceil m/3r\rceil\geq m/3r$. Our approximation factor is $r= 2^k/9$; hence, $d\geq m/3r=3m/2^k$ and $2^k/3\euler r=3/\euler$. Since clearly $d\leq m$, the factor $1/d^{k-1}$ in Eq. \eqref{eq:hyper} is not greater than $m^{-k}$, and we have a lower bound \[ t\geq \left(\frac{3}{\euler}\right)^{6m/2^k}\cdot d^{-k} \geq 2^{0.8 m/2^k - k\log m }\,. \] From our assumption $k\leq \log \sqrt{m}$, we have $m/2^k\geq \sqrt{m}\gg k\log m$, and the desired lower bound $t\geq 2^{\Omega(m/2^k)}\geq 2^{\Omega(\sqrt{m})}$ follows.

Remark: The reason why this argument fails for bipartite graphs ($k=2$) and the factor $r=2$ is numerical: in this case, we would only have $2^k/3\euler r=2/3\euler \lt 1$ in Eq. \eqref{eq:hyper}, and the lower bound would be trivial. On the other hand, the argument used for larger values of $k$ is quite "brutal," and it could apparently be possible to find non-trivial upper bounds on $|\f_{\R}|$ also in the case $k=2$ by going deeper in the special properties of perfect matchings in $K_{m,m}$. $\Box$


Footnotes:

(1)   Jump back ☝


References:

  1. S. Jukna and H. Seiwert: Approximation limitations of pure dynamic programming, SIAM J. Comput., 49:1 (2020) 170-207.    arxiv

⇦   Back to the comments page